物理/8章の付録

提供: Internet Web School

(版間での差分)
( 指数関数)
( 指数関数)
234 行: 234 行:
$f_{a}(x)\triangleq a^x  \qquad (x は任意の実数)$ という関数を'''指数関数'''と呼ぶ。<br/><br/>
$f_{a}(x)\triangleq a^x  \qquad (x は任意の実数)$ という関数を'''指数関数'''と呼ぶ。<br/><br/>
'''命題1'''<br/>
'''命題1'''<br/>
-
(1)$a$ が 1 より大きい実数の時,$f_{a}(x)= a^x $ は単調増加 <br/>
+
(1)$a$ が 1 より大きい実数の時, $f_{a}(x)= a^x $ は単調増加 <br/>
-
(2)$a$ が 1 より小さい実数の時,$f_{a}(x)= a^x $ は単調減少<br/>
+
(2)$a$ が 1 より小さい実数の時, $f_{a}(x)= a^x $ は単調減少<br/>
-
(3)$a = 1$ のとき、$f_{1}(x)= 1^x  \equiv 1$<br/><br/>
+
(3)$a = 1$ のとき、 $f_{1}(x)= 1^x  \equiv 1$<br/><br/>
'''命題2'''<br/>
'''命題2'''<br/>
指数関数 $f_{a}(x)= a^x $ は<br/>
指数関数 $f_{a}(x)= a^x $ は<br/>
242 行: 242 行:
(2) 定義域は実数全体${\bf R}$<br/>
(2) 定義域は実数全体${\bf R}$<br/>
(3) $a \neq 1 $ の時、<br/>
(3) $a \neq 1 $ の時、<br/>
-
値域 $¥mathscr{R}\triangleq \{a^x |x \in {\bf R} \}$ は$(0,\infty)\triangleq \{x|x \in {\bf R},x \gt 0 \}$ に等しく、<br/>
+
値域 $\mathcal{R}\triangleq \{a^x |x \in {\bf R} \}$ は$(0,\infty)\triangleq \{x|x \in {\bf R},x \gt 0 \}$ に等しく、<br/>
-
一対一関数($ x \neq x' \imply a^x \neq a^{x'}$)である。<br/>
+
一対一関数($ x \neq x' \rightarrow  a^x \neq a^{x'}$)である。<br/>
'''命題3'''<br/>
'''命題3'''<br/>
e をネイピア数とすると<br/>
e をネイピア数とすると<br/>

2018年1月6日 (土) 03:54時点における版

目次

 8章の付録

 問の解答

 問

$\lim_{n\to \infty}(1+\frac{1}{n})^n$ が存在し、2より大きく3以下であることを証明する。
(1)準備; 2項定理;を用いた展開
$a_n\triangleq (1+\frac{1}{n})^{n} \qquad (n は自然数)$ とおく。
すると、 $2 \leq a_1=1+\frac{1}{1}=2\quad \lt a_2=(1+\frac{1}{2})^{2} =2\frac{1}{4}$である。
以下に、数列 $\{a_n\}_{n=1}^{\infty}$ が単調増大で、有界(2より大、3より小)である事を示す。
するとテキストの定理により、この数列は2より大きく、3以下のある実数に収束することが分かる。
nが3以上の自然数の時は、$a_n$を2項定理を用いて展開すると
  $a_n=(1+\frac{1}{n})^{n}=\sum_{m=0}^{n}{}_n\mathrm{C}_{m}1^{n-m}(\frac{1}{n})^m \qquad \qquad (1)$
  ここで ${}_n\mathrm{C}_{m}$ は、n個のものからm個取り出す取り出し方の総数で、
mが1以上でn 以下の自然数の時は
${}_n\mathrm{C}_{m}=\frac{n!}{m!(n-m)!} \qquad \qquad (2)$
ここで、m が1以上の自然数の時は $ m!\triangleq 1\cdot 2\cdot 3 \cdots (m-1)\cdot m$
mが零の時は $\quad 0!\triangleq 1 $ と定義。

すると、
${}_n\mathrm{C}_{0}=\frac{n!}{0!n!}=1\qquad \qquad (3)$ 
$m \geq 1$のとき、${}_n\mathrm{C}_{m} =\frac{n!}{m!(n-m)!}=\frac{n\cdot (n-1)\cdot (n-2) \cdots \Bigl(n-(m-1)\Bigr) }{m!} \qquad (4)$
式(1)に式(2)を代入し,式(3)、(4)を利用して計算すると
$a_n = 1+\sum_{m=1}^{n}\frac{n(n-1)(n-2)\cdots \Bigl(n-(m-1)\Bigr)}{m!}1^{n-m}(\frac{1}{n})^m $
$=2+\sum_{m=2}^{n}\frac{1(1-\frac{1}{n})(1-\frac{2}{n})\cdots (1-\frac{m-1}{n})}{m!}\qquad \qquad (5)$
ここで、n より小さい全ての自然数 i に対して
$0 \lt 1-\frac{i}{n} \lt 1 $ なので、
$ 2 \lt a_n \lt 2+\sum_{m=2}^{n}\frac{1}{m!} \qquad \qquad \qquad (6)$

(2)すべての2以上の自然数 n に関して、
$ 2 \lt a_n \lt 3 \qquad \qquad \qquad (7)$
であることを示す。
式(6)から
$2\lt a_n$,
$a_n \lt 2+\sum_{m=2}^{n}\frac{1}{m!} \qquad \qquad (8)$
右辺の m は2以上の自然数なので、
$\frac{1}{m!} \leq \frac{1}{(m-1)m}=\frac{1}{m-1}-\frac{1}{m}$
である。故に、
$a_n \lt 2+\sum_{m=2}^{n}(\frac{1}{m-1}-\frac{1}{m})=2+(1-\frac{1}{n})=3-\frac{1}{n}\lt 3$

(3)数列 $\{a_n\}_{n=1}^{\infty}$ は単調増加
$n \geq 2$ の時、常に $a_n \lt a_{n+1}$ を示せばよい。
式(5)を利用すると(注参照)、
$a_{n+1}=2+\sum_{m=2}^{n+1}\frac{1(1-\frac{1}{n+1})(1-\frac{2}{n+1})\cdots (1-\frac{m-1}{n+1})}{m!}$
すると、
$a_{n+1} - a_n = \sum_{m=2}^{n+1}\frac{1(1-\frac{1}{n+1})(1-\frac{2}{n+1})\cdots (1-\frac{m-1}{n+1})}{m!} - \sum_{m=2}^{n}\frac{1(1-\frac{1}{n})(1-\frac{2}{n})\cdots (1-\frac{m-1}{n})}{m!} $
$\quad$ 右辺の第一項の和を2つに分けると、
$= \frac{1(1-\frac{1}{n+1})(1-\frac{2}{n+1})\cdots (1-\frac{n}{n+1})}{m!}$
$\quad + \sum_{m=2}^{n}\frac{1(1-\frac{1}{n+1})(1-\frac{2}{n+1})\cdots (1-\frac{m-1}{n+1})}{m!} - \sum_{m=2}^{n}\frac{1(1-\frac{1}{n})(1-\frac{2}{n})\cdots (1-\frac{m-1}{n})}{m!}$

$ = \frac{ 1(1-\frac{1}{n+1})(1-\frac{2}{n+1})\cdots (1-\frac{n}{n+1}) }{m!}$
$\quad + \sum_{m=2}^{n}\frac{ 1(1-\frac{1}{n+1})(1-\frac{2}{n+1})\cdots (1-\frac{m-1}{n+1}) -1(1-\frac{1}{n})(1-\frac{2}{n})\cdots (1-\frac{m-1}{n})}{m!}$
上の式で、全ての$i\in \{1,2,,,,n\}$に対して,$(1-\frac{i}{n+1})\gt 0$と$(1-\frac{i}{n+1})\gt (1-\frac{i}{n})$ なので、
$a_{n+1} - a_n \gt 0$

(注)式(3)のnに n+1 を代入すればよい。

 ネイピア数 e について

定義;$e\triangleq \lim_{}(1+\frac{1}{n})^n$ネイピア数と呼ぶ。
命題1
(1)$ 2 \lt e \leq 3$
(2)$e=\sum_{m=0}^{\infty}\frac{1}{m!} \qquad ただし、0!\triangleq 1,\quad m!\triangleq 1\cdot 2\cdot 3\cdots (m-1)\cdot m \qquad \qquad (9)$

 三角関数の微分

 準備 

次の命題が、三角関数の微分を求めるうえで中心的役割を果たす。 
命題2 
$\lim_{\theta\to 0,\theta\neq 0}\frac{\sin \theta}{\theta}=1$
証明
まず、$\theta$ を正に保ちながら零に近づける場合を考える。
すると、$ 0 \lt \theta \lt \pi/2 $ と考えて良い。
点Oを中心にし、半径1の円を考え、円周上に一点Aをさだめる。
図のように、円周上の点Bを、線分OBが直線OAとなす角がx(ラジアン)となるようにとる。

図から$\triangle{OAB} \subset 扇形OAB \subset \triangle{OAP} $
$\quad$ ここで、点PはAを通り線分OAと垂直な直線と半直線OBの交点。
すると、
$\triangle{OAB}の面積 \lt 扇形OAB の面積 \lt \triangle{OAP}の面積$
ここで、$\triangle{OAB}の面積=\frac{1\cdot \sin{\theta}}{2},\quad 扇形OAB の面積=\pi\cdot 1^{2}\cdot \frac{\theta}{2\pi}\quad \triangle{OAP}の面積=\frac{1\cdot \tan{\theta}}{2}$なので、
$\frac{\sin{\theta}}{2} \lt \frac{\theta}{2}\lt \frac{\tan{\theta}}{2}=\frac{\sin{\theta}}{2\cos{\theta}}\qquad $各項を2倍すると、
$\sin{\theta}\lt \theta \lt \frac{\sin{\theta}}{\cos{\theta}}$
$\quad$ここで $\sin{\theta}\gt 0$ なので、これで上式の各項を割ると、
$1 \lt \frac{\theta}{\sin{\theta}} \lt \frac{1}{\cos{\theta}}$
$1 \gt \frac{\sin{\theta}}{\theta} \gt \cos{\theta}$
故に、極限の性質から
$1 \geq \lim_{\theta\to 0,\theta\neq 0}\frac{\sin \theta}{\theta} \geq \lim_{\theta\to 0,\theta\neq 0}\cos{\theta}=1$
これより、$\lim_{\theta\to 0,\theta\neq 0}\frac{\sin \theta}{\theta}=1$ が得られる。

定理1 三角関数の微分
(1)$\frac{d}{d\theta}\sin{\theta}=\cos{\theta}$
(2)$\frac{d}{d\theta}\cos{\theta}=-\sin{\theta}$
証明
(1); $\frac{d}{d\theta}\sin{\theta} \triangleq \lim_{h\to 0,h\neq 0}\frac{\sin (\theta+h)-\sin \theta}{h}$
ここで、
$\sin (\theta+h)-\sin \theta = \sin \bigl((\theta + \frac{h}{2})+\frac{h}{2}\bigr) - \sin \bigl((\theta + \frac{h}{2})-\frac{h}{2}\bigr)$
サイン関数の加法定理を適用すると
$=\sin (\theta + \frac{h}{2})\cos \frac{h}{2} + \cos (\theta + \frac{h}{2})\sin \frac{h}{2} - \Bigl( \sin (\theta + \frac{h}{2})\cos \frac{h}{2} - \cos (\theta + \frac{h}{2})\sin \frac{h}{2} \Bigr) = 2\cdot \cos (\theta + \frac{h}{2})\sin \frac{h}{2}$
故に、 $\frac{d}{d\theta}\sin{\theta} \triangleq \lim_{h\to 0,h\neq 0}\frac{\sin (\theta+h)-\sin \theta}{h} = \lim_{h\to 0,h\neq 0}\frac{2\cdot \cos (\theta + \frac{h}{2})\sin \frac{h}{2}}{h}=\lim_{h\to 0,h\neq 0}\cos (\theta + \frac{h}{2})\frac{\sin \frac{h}{2}}{h/2}$
$=\lim_{h\to 0,h\neq 0}\cos (\theta + \frac{h}{2})\lim_{h\to 0,h\neq 0}\frac{\sin \frac{h}{2}}{h/2}$
$\quad $ ここで、
$\quad \lim_{h\to 0,h\neq 0}\cos (\theta + \frac{h}{2}) = \cos \theta$
$\quad \lim_{h\to 0,h\neq 0}\frac{\sin \frac{h}{2}}{h/2} = 1 \quad (上の命題2より)$
$\quad $なので、
$=\cos \theta$

指数関数と対数関数

 実数の累乗

a を任意の実数、n を2以上の自然数とする。
$a^1=a,\quad a^2=a\cdot a,\quad a^3=a^2\cdot a=a\dot a\cdot a \cdots a^n=a^{n-1}\cdot a, \cdots$
を総称して、a の累乗と呼ぶ。
$a^n$ を、a の n 乗 、n をその指数と呼ぶ。
命題1
a,b を任意の実数、m,nを任意の自然数とすると、
(1) $a^{m}a^{n} = a^{m+n} $
(2) $(a^{m})^n =a^{m n} $
(3) $(ab)^n = a^n b^n $

そこで次の累乗に関する計算規則を定義する。

累乗に関する計算規則
a,b を任意の正の実数、$\alpha,\quad \beta$を指数を表わす数とすると、
(1) $a^{\alpha}a^{\beta} = a^{\alpha+\beta} \qquad \qquad \qquad (累乗規則1)$
(2) $(a^{\alpha})^\beta =a^{\alpha \beta} \qquad \qquad \qquad (累乗規則2)$
(3) $(ab)^\beta = a^\beta b^\beta \qquad \qquad \qquad (累乗規則3)$

命題2
(1)a を1より小さい正の実数とすると、数列 $\{a^{n}\}_{n=1}^{\infty}$ は単調減少し、零に収束。
(2)a を1より大きい正の実数とすると、数列 $\{a^{n}\}_{n=1}^{\infty}$ は単調増加し、いくらでも大きくなる(無限大に発散)。

これより、累乗に関する3つの規則が、そのまま成り立つようにしながら、指数を実数まで拡げよう。

 指数の整数への拡張

まず指数を、累乗に関する3つの規則が成り立つようにしながら、整数に拡張する。
累乗の定義から、
$a \neq 0$ の時は、任意の自然数m、nに対し、
$a^m \div a^n = a^{m-n} \qquad (m\gt n) \qquad \qquad (1)$
$\qquad \qquad = 1 \qquad (m = n)\qquad \qquad \qquad(2)$
$\qquad \qquad = \frac{1}{a^{n-m}} \qquad (m \lt n)\qquad \qquad (3)$
であることが分かる。
これを一つの式 $a^{m-n}$ で表わせるように、a の指数を取決めたい。
そのためには、指数が零の時、$a^0 \triangleq 1$、
指数 $m-n$ が負数の時 $a^{m-n} \triangleq \frac{1}{a^{n-m}}$ 
と定義すればよい。
言い換えると、$a (\neq 0) $ の指数nが 零と負の整数のとき、
$a^0 \triangleq 1, \qquad a^n \triangleq \frac{1}{a^{-n}} \qquad (n\lt 0)\qquad \qquad (4)$
と定義する。
すると、指数が整数の時、3つの累乗規則を満たすことは、容易に確かめられる。

 指数の有理数への拡張

a を任意の正の実数、 $\frac{m}{n}$ を任意の有理数のとき、
a の有理数乗 $a^{\frac{m}{n}}$ を、次のような計算規則を満たすように定義しよう。

$a^{\frac{m}{n}} = a^{\frac{-m}{-n}}$ なので、指数n を任意の自然数(正の整数)、 m を任意の整数と仮定してよい。
累乗規則(2)を満たすように定義するには、
$(a^{\frac{m}{n}})^{n} = (a^{\frac{m}{n}})^{\frac{n}{1}} = a^{m}$ 
でなければならない。
そこで、 n乗すると、$a^{m}$ となる正の実数を$a^{\frac{m}{n}}$ と決めることが 自然であろう。
最初に、この定義できちんと正の実数が一つだけ決まることを証明しよう。

命題3
a を任意の正の実数、m,n を任意の整数とする。
すると、n乗すると $a^{m}$ になる正の実数 b が存在し、ただ一つに限る。
証明;
$ f(x) \triangleq x^n $ という、零と正の実数の上で定義された、関数を考える。
この関数はxが増加するにつれて、連続的に、零から正の無限大に狭義に単調に増加(注参照)していく。
一方、$a^{m}$ は必ず正の実数である。
そのため、xが零のときは、 $ f(x) \lt a^{m}$ であり、
xを少し増加させても、同じ関係が成り立つ。
関数は単調に零から∞まで増加していくので、
ある正の実数(b と書こう)まで、$ f(x) \lt a^{m}$ であり
b以上の実数xに対しては、$ f(x) \geq a^{m}$ となることが分かる。
関数値は、x の変化につれて連続に変化するので$ f(b) = a^{m}$ である。
関数fが狭義単調増加なので,b以外の正の実数xでは、$ f(x) \neq a^{m}$ である。
(証明終り)
(注) 関数fが狭義単調増加とは、$ x \lt y \Rightarrow f(x) \lt f(y) $ を満たすこと。

命題4
任意の正の実数 $a$ にたいして、その有理数乗を上記のように定義すると
3つの累乗規則 (1)~(3) が成り立つ。
証明; ① 累乗規則(1)が成り立つことを示す。
2個の有理数の指数を 自然数$n,\quad \tilde{n}$と整数$m,\quad \tilde{m}$ を用いて、
$\alpha = \frac{m}{n},\quad \beta = \frac{\tilde{m}}{\tilde{n}}$と表現する。
すると、累乗規則(1)は、次のように表される。
$a^{\frac{m}{n}}a^{\frac{\tilde{m}}{\tilde{n}}} = a^{\frac{m}{n}+\frac{\tilde{m}}{\tilde{n}}} $
この左辺を $b \triangleq a^{\frac{m}{n}}a^{\frac{\tilde{m}}{\tilde{n}}}$,
右辺を $c \triangleq a^{\frac{m}{n}+\frac{\tilde{m}}{\tilde{n}}}$ とおく。
$b^{n \tilde{n}} = c^{n \tilde{n}}  \qquad \qquad \qquad (A)$ 
であることを示せば、$b = c$ が得られ,
累乗規則(1)が成立することが分かる。
まず左辺を考える。
$b^{n \tilde{n}} = (a^{\frac{m}{n}}a^{\frac{\tilde{m}}{\tilde{n}}})^{n \tilde{n}}$
指数が自然数の累乗規則(3)から
$ = (a^{\frac{m}{n}})^{n \tilde{n}}(a^{\frac{\tilde{m}}{\tilde{n}}})^{n \tilde{n}}$
指数が自然数の累乗規則(2)から
$= \Bigl((a^{\frac{m}{n}})^{n}\Bigr)^{\tilde{n}}\Bigl((a^{\frac{\tilde{m}}{\tilde{n}}})^{\tilde{n}}\Bigr)^{n}$
実数の有理数乗の定義から、
$ = (a^{m})^{\tilde{n}}(a^{\tilde{m}})^{n}$
指数が整数の累乗規則(2)から
$ = a^{m \tilde{n}}a^{\tilde{m}n}$
指数が整数の累乗規則(1)から
$ = a^{m \tilde{n} + \tilde{m}n}$
故に、$b^{n \tilde{n}} = a^{m \tilde{n} + \tilde{m}n}$
次に、右辺を考える。
$c^{n \tilde{n}} = (a^{\frac{m}{n}+\frac{\tilde{m}}{\tilde{n}}})^{n \tilde{n}}$

$ = (a^{ \frac{m\tilde{n}+n\tilde{m}}{n\tilde{n}}})^{n \tilde{n}}$
実数の有理数乗の定義から、
$ = a^{m\tilde{n} + n\tilde{m}} = b^{n \tilde{n}}$
これで、式(A)が示され、累乗規則(1)が成り立つことが証明できた。
 

② 累乗規則(2)が成り立つことを示す。
③ 累乗規則(3)が成り立つことを示す。
証明終わり。

指数が有理数の場合,命題2は次のように拡張出来きる。

命題5
有理数全体の上で定義される関数
$f_a(\alpha) \triangleq a^{\alpha} \qquad (\alpha は有理数)$を考える。
(1)a が1より小さい正の実数ととき、
$f_a$は単調減少し、
$\lim_{\alpha \to \infty}f_a(\alpha) = 0\quad \lim_{\alpha \to -\infty}f_a(\alpha) = \infty $
2)a を1より大きい正の実数とすると、
$f_a$は単調増大で
$\lim_{\alpha \to \infty}f_a(\alpha) = \infty \quad \lim_{\alpha \to -\infty}f_a(\alpha) = 0 $
3)a = 1 のとき、$f_a \equiv 1$
証明
証明終わり

 指数の実数への拡張

任意の実数 $\alpha$ に対して $a^{\alpha}$ を次のように定義する。
定義
$\{\alpha_n\}_{n=1}^{\infty}$ を $\alpha$ に収束する有理数の単調増加数列とするとき、
$a^{\alpha} \triangleq \lim_{n \to \infty}a^{\alpha_{n}}$ 
この定義により、唯一の実数が必ず定まることが次のようにして分かる。
命題6
① 上記の有理数の単調増加列 $\{\alpha_{n}\}_{n=1}^{\infty}$ は収束する。
② $\alpha$ に収束する、別の有理数の単調増加数列 $\{\beta_n\}_{n=1}^{\infty}$ に対して、$\lim_{n \to \infty}a^{\beta_{n}} = \lim_{n \to \infty}a^{\alpha_{n}}$ 
証明
証明終わり

 指数関数

aを任意の正の実数とする。
定義
$f_{a}(x)\triangleq a^x \qquad (x は任意の実数)$ という関数を指数関数と呼ぶ。

命題1
(1)$a$ が 1 より大きい実数の時, $f_{a}(x)= a^x $ は単調増加
(2)$a$ が 1 より小さい実数の時, $f_{a}(x)= a^x $ は単調減少
(3)$a = 1$ のとき、 $f_{1}(x)= 1^x \equiv 1$

命題2
指数関数 $f_{a}(x)= a^x $ は
(1) 連続である。
(2) 定義域は実数全体${\bf R}$
(3) $a \neq 1 $ の時、
値域 $\mathcal{R}\triangleq \{a^x |x \in {\bf R} \}$ は$(0,\infty)\triangleq \{x|x \in {\bf R},x \gt 0 \}$ に等しく、
一対一関数($ x \neq x' \rightarrow a^x \neq a^{x'}$)である。
命題3
e をネイピア数とすると
$ e^x = \sum_{n=0}^{\infty}\frac{x^n}{n!} \quad (xは任意の実数) \qquad \qquad (1)$
証明
$\quad$ 補助命題
$\quad$ 実数全体の上の関数 Exp を
$\quad$ $Exp(x) \triangleq \sum_{n=0}^{\infty}\frac{x^n}{n!}$ 
$\quad$ で定義する。
$\quad$ この関数は次の性質をもつ。
$\quad$ (1) $Exp(1) = e$
$\quad$ (2) $ Exp(x+y)=Exp(x)Exp(y) \quad (x,yは任意の実数)$
$\quad$ (3) 任意の有理数 $\alpha$ に対して $Exp(\alpha) = e^{\alpha}$
$\quad$ (4) 関数 $Exp$ は微分可能、従って連続関数。
$\quad$ 補助命題の証明
$\quad$ 補助命題の証明終り。
証明終わり。

 対数関数

指数関数と対数関数の微分

個人用ツール